2007 AMC 12A Problems/Problem 2

Revision as of 11:41, 9 September 2007 by Azjps (talk | contribs) (Solution: strange, the equation is disappearing for me)

Problem

An aquarium has a rectangular base that measures 100 cm by 40 cm and has a height of 50 cm. It is filled with water to a height of 40 cm. A brick with a rectangular base that measures 40 cm by 20 cm and a height of 10 cm is placed in the aquarium. By how many centimeters does the water rise?

$\mathrm{(A)}\ 0.5\qquad \mathrm{(B)}\ 1\qquad \mathrm{(C)}\ 1.5\qquad \mathrm{(D)}\ 2\qquad \mathrm{(E)}\ 2.5$

Solution

The water has volume $100 \cdot 40 \cdot 40=160000$. The brick has volume 8000. The water and the brick combined have a volume of 168000. The water rises $\frac{168000}{4000}-\frac{160000}{4000}=42-40=2$ cm.

See also

2007 AMC 12A (ProblemsAnswer KeyResources)
Preceded by
Problem 1
Followed by
Problem 3
1 2 3 4 5 6 7 8 9 10 11 12 13 14 15 16 17 18 19 20 21 22 23 24 25
All AMC 12 Problems and Solutions